Communities

Writing
Writing
Codidact Meta
Codidact Meta
The Great Outdoors
The Great Outdoors
Photography & Video
Photography & Video
Scientific Speculation
Scientific Speculation
Cooking
Cooking
Electrical Engineering
Electrical Engineering
Judaism
Judaism
Languages & Linguistics
Languages & Linguistics
Software Development
Software Development
Mathematics
Mathematics
Christianity
Christianity
Code Golf
Code Golf
Music
Music
Physics
Physics
Linux Systems
Linux Systems
Power Users
Power Users
Tabletop RPGs
Tabletop RPGs
Community Proposals
Community Proposals
tag:snake search within a tag
answers:0 unanswered questions
user:xxxx search by author id
score:0.5 posts with 0.5+ score
"snake oil" exact phrase
votes:4 posts with 4+ votes
created:<1w created < 1 week ago
post_type:xxxx type of post
Search help
Notifications
Mark all as read See all your notifications »
Q&A

The Fourier transform of $1/p^3$

+2
−0

Take the following Fourier transform conventions $$ \tilde{f}(\mathbf{p}) =\int f(\mathbf{x}) \ e^{-i\mathbf{p}\cdot\mathbf{x}} \ d^3x $$ $$ f(\mathbf{x}) =\int \tilde{f}(\mathbf{p})\ e^{i\mathbf{p}\cdot\mathbf{x}} \ \frac{d^3p}{(2\pi)^3}. $$ In physics, it is useful to compute the transform of $1/p^3$ $$ I=\int \frac{1}{p^3}\ e^{i\mathbf{p}\cdot\mathbf{x}} \ \frac{d^3p}{(2\pi)^3}. $$ But I have no idea what it even mean since the integral is not well-defined.

History
Why does this post require moderator attention?
You might want to add some details to your flag.
Why should this post be closed?

2 comment threads

Welcome to Math Codidact. Are you the author of the question that was posted at SE, or are you copyi... (1 comment)
X-post https://math.stackexchange.com/questions/3723136/the-fourier-transform-of-1-p3 (1 comment)

1 answer

+1
−0

In short, in $\mathbb{R}^d$, if I define the Fourier transform as $\mathcal{F}(f)(x) = \int_{\mathbb{R}^d} f(y) \,e^{-2iπxy}\,\mathrm{d}y$ the result is $$ \boxed{\mathcal{F}\left(\frac{1}{\omega_d,|x|^d}\right) = \frac{\psi(d/2)-\gamma}{2} - \ln(|πx|)} $$ where $\omega_d = \frac{2\,\pi^{d/2}}{\Gamma(d/2)}$ is the size of the unit sphere (so $\omega_3 = 4\pi$), $\gamma$ is Euler–Mascheroni constant and $\psi$ is the digamma function. Since $\psi(3/2) = 2-\gamma-\ln(4)$, we deduce that in dimension $3$ $$ \mathcal{F}\left(\frac{1}{4\pi,|x|^3}\right) = 1-\gamma -\ln(|2πx|) $$ which with your Fourier transform convention gives $$ \boxed{\tilde{\mathcal{F}}\left(\frac{1}{|x|^3}\right) = \frac{1-\gamma -\ln(|x|)}{2\pi^2}.} $$


Now the details. So first, what is the meaning of $\frac{1}{|x|^d}$? One can define the following distribution $$ v_d := \mathrm{pf}\left(\frac{1}{|x|^d}\right) := \mathrm{div}\left(\frac{x\ln(|x|)}{|x|^d}\right) $$ where the derivative is taken in the sense of distributions. One easily verifies that the distribution $v_d$ coincides on $\Bbb R^d\setminus\{0\}$ with the function $$ v_d(x) = \frac{1}{|x|^d} \text{ for any } x≠ 0. $$ One also notices that it is a tempered distribution as the derivative of a function in $L^1 + L^\infty$.


Let $u_d = \frac{x\ln(|x|)}{|x|^d}$. Multiplying by a test function $\varphi\in C^\infty_c$, one gets for any $\lambda>0$ $$ \begin{align*} \langle v_d,\varphi\rangle &= -\int_{\mathbb{R}^d} u_d\cdot\nabla\varphi \newline &= -\int_{|x|\leq\lambda} u_d\cdot\nabla(\varphi(x)-\varphi(0)) - \int_{|x|>\lambda} u_d\cdot\nabla \varphi \end{align*} $$ which by integration by parts yields $$ \begin{align}\tag{1} \langle v_d,\varphi\rangle &= \int_{|x|\leq\lambda} \frac{\varphi(x)-\varphi(0)}{|x|^d},\mathrm{d}x \newline &\quad+ \int_{|x|>\lambda} \frac{\varphi(x)}{|x|^d} ,\mathrm{d}x + \omega_d \ln(\lambda) ,\varphi(0). \end{align} $$ One can take $\lambda = 1$ to get $$ \begin{align*} \langle v_d,\varphi\rangle &= \int_{|x|\leq 1} \frac{\varphi(x)-\varphi(0)}{|x|^d},\mathrm{d}x + \int_{|x|> 1} \frac{\varphi(x)}{|x|^d} ,\mathrm{d}x. \end{align*} $$ But with the formula (1) with $\lambda\neq 1$, one can also compute easily the distribution $v_d(\lambda x)$ defined by $$ \langle v_d(\lambda x),\varphi(x)\rangle = \frac{1}{|\lambda|^d}\langle v_d(x),\varphi(x/\lambda)\rangle $$ using Equation (1) with $\varphi(x)$ replaced by $\varphi(x/\lambda)$. Doing the change of variable $x/\lambda \to x$ yields $$ v_d(\lambda,\cdot) = \frac{1}{|\lambda|^d},v_d + \omega_d\frac{\ln(\lambda)}{|\lambda|^d},\delta_0. $$ Therefore, we can now use the scaling properties of the Fourier transform to get for any $r=1/\lambda>0$ $$ \begin{align*} (\mathcal{F}(v_d))(r\tilde{x}) &= r^{-d} (\mathcal{F}(v_d(y/r)))(\tilde{x}) \newline&= \mathcal{F}(v_d-\omega_d\ln(r)\delta_0)(\tilde{x}) \newline&= \mathcal{F}(v_d)(\tilde{x}) -\omega_d\ln(r). \end{align*} $$ Taking $\tilde{x} = \frac{x}{|x|}$ and $r=|x|$ gives $$ \boxed{\mathcal{F}(v_d)(x) = C_d -\omega_d\ln(|x|)} $$ where $C_d = \mathcal{F}(v_d)(\tilde{x})$ is a constant since the Fourier transform of a radial function is radial.


If you want to know the constant $C_d$, the usual trick is to multiply by a Gaussian and use the fact that we know the Fourier transform of a Gaussian. Here remark first that by the Fourier inversion theorem we have $$ \mathcal{F}(\ln(|x|)) = C_d, \delta_0 - \frac{v_d}{\omega_d} $$ Therefore $$ \begin{align*} C_d - \langle\mathcal{F}(\ln(|x|)), e^{-\pi|x|^2}\rangle &= \frac{1}{\omega_d}\langle v_d, e^{-\pi|x|^2}\rangle \newline &= \int_0^1 \frac{e^{-\pi r^2}-1}{r},\mathrm{d} r + \int_1^\infty \frac{e^{-\pi r^2}}{r},\mathrm{d} r \newline &= \int_0^\pi \frac{e^{-t}-1}{2,t},\mathrm{d} t + \int_\pi^\infty \frac{e^{-t}}{2,t},\mathrm{d} t \newline &= \frac{-\ln(\pi)}{2} + \int_0^\pi \frac{\ln(t),e^{-t}}{2},\mathrm{d} t + \int_\pi^\infty \frac{\ln(t),e^{-t}}{2},\mathrm{d} t \newline &= \frac{-\gamma - \ln(\pi)}{2} \end{align*} $$ where $\gamma = -\Gamma'(1) = -\psi(1)$ and I did a polar change of variable and the change $t = πr^2$ and then integrations by parts. But since $\mathcal{F}(e^{-\pi|x|^2}) = e^{-\pi|x|^2}$, we can also compute $$ \begin{align*} \langle\mathcal{F}(\ln(|x|)), e^{-\pi|x|^2}\rangle &= \int_{\mathbb{R}^d} \ln(|x|), e^{-\pi|x|^2},\mathrm{d} x \newline &= \omega_d \int_0^\infty \ln(r),e^{-\pi r^2} r^{d-1},\mathrm{d} r \newline &= \frac{1}{2,\Gamma(d/2)} \int_0^\infty (\ln(t)-\ln(\pi)),e^{-t}, t^{d/2-1},\mathrm{d} t \newline &= \frac{1}{2} ,\big(\psi(d/2) - \ln(\pi)\big). \end{align*} $$ with the same changes of variable. We deduce that $C_d = \frac{\psi(d/2)-\gamma}{2} - \ln(\pi)$. Tell me if you spot any errors!

History
Why does this post require moderator attention?
You might want to add some details to your flag.

0 comment threads

Sign up to answer this question »